Epsilon-delta Proof: Show f(x)=sqrt(x) is Cont. @ x=4

  • Thread starter Thread starter GBR
  • Start date Start date
  • Tags Tags
    Proof
Click For Summary
The discussion focuses on using an epsilon-delta proof to demonstrate that the function f(x) = sqrt(x) is continuous at x = 4. A key point raised is the challenge of relating the epsilon (ε) to the delta (δ) in the proof, particularly in establishing the correct inequalities. The initial attempt incorrectly assumes that |sqrt(x) - sqrt(4)| can be directly equated to |x - 4|, which is not valid. A suggestion is made to work backwards from the desired epsilon inequality to derive a suitable bound for |x - a|. The conversation highlights the importance of correctly manipulating inequalities in epsilon-delta proofs.
GBR
Messages
3
Reaction score
0

Homework Statement



Use an epsilon-delta proof to show that f(x) = sqrt(x) is continuous in x = 4.

Homework Equations


The Attempt at a Solution



|f(x) - f(4)| = |sqrt(x) - sqrt(4)| < ε
= |x-4| < ε²

And now what? I'm not even sure I'm on the right path. My biggest problem during these proofs is choosing an ε and relating it to |x-a| < δ to complete the proof. Please bear with me; I recently went back to school as a physics undergrad, and realize my math-fu is destroyed after six years as a journalist :)
 
Physics news on Phys.org
This will work for sufficiently small ε. We have
|\sqrt{x}-2|=\frac{|\sqrt{x}+2|}{|\sqrt{x}+2|}|\sqrt{x}-2|=|x-4|/|\sqrt{x}+2|&lt;\frac{\epsilon^2}{|\sqrt{x}+2|}.
If, for example, ε<1, this is less than ε.
My strategy is to start with what you want to get (epsilon inequality) and work it backwards to get bound for |x-x0|.
 
GBR said:

Homework Statement



Use an epsilon-delta proof to show that f(x) = sqrt(x) is continuous in x = 4.


Homework Equations





The Attempt at a Solution



|f(x) - f(4)| = |sqrt(x) - sqrt(4)| < ε
= |x-4| < ε²

And now what? I'm not even sure I'm on the right path. My biggest problem during these proofs is choosing an ε and relating it to |x-a| < δ to complete the proof. Please bear with me; I recently went back to school as a physics undergrad, and realize my math-fu is destroyed after six years as a journalist :)

You can't go from this inequality |sqrt(x) - sqrt(4)| < ε to this one |x-4| < ε². The reason is that |sqrt(x) - sqrt(4)|² is not equal to |x - 4|.
 
Question: A clock's minute hand has length 4 and its hour hand has length 3. What is the distance between the tips at the moment when it is increasing most rapidly?(Putnam Exam Question) Answer: Making assumption that both the hands moves at constant angular velocities, the answer is ## \sqrt{7} .## But don't you think this assumption is somewhat doubtful and wrong?

Similar threads

  • · Replies 31 ·
2
Replies
31
Views
2K
  • · Replies 6 ·
Replies
6
Views
2K
  • · Replies 9 ·
Replies
9
Views
4K
  • · Replies 8 ·
Replies
8
Views
2K
  • · Replies 12 ·
Replies
12
Views
2K
  • · Replies 22 ·
Replies
22
Views
4K
  • · Replies 8 ·
Replies
8
Views
7K
  • · Replies 5 ·
Replies
5
Views
3K
  • · Replies 5 ·
Replies
5
Views
2K
  • · Replies 2 ·
Replies
2
Views
3K